Applying the Euler-Lagrange equations to Maxwell's TheoryDerivation of Maxwell's equations from field tensor lagrangianHow the boundary term in the variation of the action vanishesCoefficient matrix of quadratic LagrangianWhich transformations *aren't* symmetries of a Lagrangian?Problems while doing $dfracpartialpartial(partial_mu phi)$ and $dfracpartialpartial(partial_mu A_mu)$Problem to find field equations with Euler-Lagrange in field theoryProblem with Lagrangian densityEuler-Lagrange for simple scalar field (Peskin & Shroeder)Derivatives in Euler-Lagrange for fieldsDeriving Euler-Lagrange for Electrodynamics LagrangianBox form of the kinetic term and Euler-Lagrange equation

The Digit Triangles

Quoting Keynes in a lecture

awk assign to multiple variables at once

Review your own paper in Mathematics

What is the English pronunciation of "pain au chocolat"?

A Trivial Diagnosis

Make a Bowl of Alphabet Soup

Creating two special characters

Pre-mixing cryogenic fuels and using only one fuel tank

Non-trope happy ending?

Doesn't the system of the Supreme Court oppose justice?

What do you call a word that can be spelled forward or backward forming two different words

Why should universal income be universal?

How to get directions in deep space?

Is there a nicer/politer/more positive alternative for "negates"?

C++ copy constructor called at return

Has the laser at Magurele, Romania reached a tenth of the Sun's power?

Is there a RAID 0 Equivalent for RAM?

Find the next value of this number series

Why do ¬, ∀ and ∃ have the same precedence?

How does electrical safety system work on ISS?

Do we have to expect a queue for the shuttle from Watford Junction to Harry Potter Studio?

What features enable the Su-25 Frogfoot to operate with such a wide variety of fuels?

What fields between the rationals and the reals allow a good notion of 2D distance?



Applying the Euler-Lagrange equations to Maxwell's Theory


Derivation of Maxwell's equations from field tensor lagrangianHow the boundary term in the variation of the action vanishesCoefficient matrix of quadratic LagrangianWhich transformations *aren't* symmetries of a Lagrangian?Problems while doing $dfracpartialpartial(partial_mu phi)$ and $dfracpartialpartial(partial_mu A_mu)$Problem to find field equations with Euler-Lagrange in field theoryProblem with Lagrangian densityEuler-Lagrange for simple scalar field (Peskin & Shroeder)Derivatives in Euler-Lagrange for fieldsDeriving Euler-Lagrange for Electrodynamics LagrangianBox form of the kinetic term and Euler-Lagrange equation













4












$begingroup$


In Prof. David Tong's notes, specifically on page 10, he gives the Lagrangian of Maxwell's theory to be



$$
mathcalL = -frac12(partial_mu A_nu)(partial^mu A^nu) + frac12(partial_mu A^mu)^2
$$



and then he computes the following



$$
fracpartialmathcalLpartial(partial_mu A_nu) = -partial_mu A_nu + (partial_rho A^rho)eta^munu.
$$



I can see how the first term in the derivative is computed but am having problems with the second term. Any help is appreciated!










share|cite|improve this question











$endgroup$











  • $begingroup$
    Possible duplicates: physics.stackexchange.com/q/3005/2451 and links therein.
    $endgroup$
    – Qmechanic
    Mar 18 at 11:09















4












$begingroup$


In Prof. David Tong's notes, specifically on page 10, he gives the Lagrangian of Maxwell's theory to be



$$
mathcalL = -frac12(partial_mu A_nu)(partial^mu A^nu) + frac12(partial_mu A^mu)^2
$$



and then he computes the following



$$
fracpartialmathcalLpartial(partial_mu A_nu) = -partial_mu A_nu + (partial_rho A^rho)eta^munu.
$$



I can see how the first term in the derivative is computed but am having problems with the second term. Any help is appreciated!










share|cite|improve this question











$endgroup$











  • $begingroup$
    Possible duplicates: physics.stackexchange.com/q/3005/2451 and links therein.
    $endgroup$
    – Qmechanic
    Mar 18 at 11:09













4












4








4


1



$begingroup$


In Prof. David Tong's notes, specifically on page 10, he gives the Lagrangian of Maxwell's theory to be



$$
mathcalL = -frac12(partial_mu A_nu)(partial^mu A^nu) + frac12(partial_mu A^mu)^2
$$



and then he computes the following



$$
fracpartialmathcalLpartial(partial_mu A_nu) = -partial_mu A_nu + (partial_rho A^rho)eta^munu.
$$



I can see how the first term in the derivative is computed but am having problems with the second term. Any help is appreciated!










share|cite|improve this question











$endgroup$




In Prof. David Tong's notes, specifically on page 10, he gives the Lagrangian of Maxwell's theory to be



$$
mathcalL = -frac12(partial_mu A_nu)(partial^mu A^nu) + frac12(partial_mu A^mu)^2
$$



and then he computes the following



$$
fracpartialmathcalLpartial(partial_mu A_nu) = -partial_mu A_nu + (partial_rho A^rho)eta^munu.
$$



I can see how the first term in the derivative is computed but am having problems with the second term. Any help is appreciated!







homework-and-exercises electromagnetism lagrangian-formalism classical-electrodynamics variational-calculus






share|cite|improve this question















share|cite|improve this question













share|cite|improve this question




share|cite|improve this question








edited Mar 18 at 7:26









Qmechanic

106k121961224




106k121961224










asked Mar 18 at 5:05









LimzyLimzy

304




304











  • $begingroup$
    Possible duplicates: physics.stackexchange.com/q/3005/2451 and links therein.
    $endgroup$
    – Qmechanic
    Mar 18 at 11:09
















  • $begingroup$
    Possible duplicates: physics.stackexchange.com/q/3005/2451 and links therein.
    $endgroup$
    – Qmechanic
    Mar 18 at 11:09















$begingroup$
Possible duplicates: physics.stackexchange.com/q/3005/2451 and links therein.
$endgroup$
– Qmechanic
Mar 18 at 11:09




$begingroup$
Possible duplicates: physics.stackexchange.com/q/3005/2451 and links therein.
$endgroup$
– Qmechanic
Mar 18 at 11:09










1 Answer
1






active

oldest

votes


















4












$begingroup$

We have $frac12 (partial_muA^mu)^2 = frac12 (partial_alpha A^alpha)(partial_betaA^beta)= frac12 (partial_alpha A_sigma) eta^sigmaalpha(partial_betaA_rho) eta^rhobeta$ so the derivative w.r.t. $partial_mu A_nu$ is



$$frac12delta_alpha^mu delta_sigma^nu eta^sigmaalpha(partial_betaA_rho) eta^rhobeta+frac12(partial_alpha A_sigma) eta^sigmaalphadelta_beta^mu delta_rho^nu eta^rhobeta= frac12 eta^munu (partial_beta A^beta)+frac12 (partial_alphaA^alpha) eta^munu = (partial_rhoA^rho)eta^munu $$



where I've freely labeled and relabeled dummy indices.






share|cite|improve this answer









$endgroup$












    Your Answer





    StackExchange.ifUsing("editor", function ()
    return StackExchange.using("mathjaxEditing", function ()
    StackExchange.MarkdownEditor.creationCallbacks.add(function (editor, postfix)
    StackExchange.mathjaxEditing.prepareWmdForMathJax(editor, postfix, [["$", "$"], ["\\(","\\)"]]);
    );
    );
    , "mathjax-editing");

    StackExchange.ready(function()
    var channelOptions =
    tags: "".split(" "),
    id: "151"
    ;
    initTagRenderer("".split(" "), "".split(" "), channelOptions);

    StackExchange.using("externalEditor", function()
    // Have to fire editor after snippets, if snippets enabled
    if (StackExchange.settings.snippets.snippetsEnabled)
    StackExchange.using("snippets", function()
    createEditor();
    );

    else
    createEditor();

    );

    function createEditor()
    StackExchange.prepareEditor(
    heartbeatType: 'answer',
    autoActivateHeartbeat: false,
    convertImagesToLinks: false,
    noModals: true,
    showLowRepImageUploadWarning: true,
    reputationToPostImages: null,
    bindNavPrevention: true,
    postfix: "",
    imageUploader:
    brandingHtml: "Powered by u003ca class="icon-imgur-white" href="https://imgur.com/"u003eu003c/au003e",
    contentPolicyHtml: "User contributions licensed under u003ca href="https://creativecommons.org/licenses/by-sa/3.0/"u003ecc by-sa 3.0 with attribution requiredu003c/au003e u003ca href="https://stackoverflow.com/legal/content-policy"u003e(content policy)u003c/au003e",
    allowUrls: true
    ,
    noCode: true, onDemand: true,
    discardSelector: ".discard-answer"
    ,immediatelyShowMarkdownHelp:true
    );



    );













    draft saved

    draft discarded


















    StackExchange.ready(
    function ()
    StackExchange.openid.initPostLogin('.new-post-login', 'https%3a%2f%2fphysics.stackexchange.com%2fquestions%2f467109%2fapplying-the-euler-lagrange-equations-to-maxwells-theory%23new-answer', 'question_page');

    );

    Post as a guest















    Required, but never shown

























    1 Answer
    1






    active

    oldest

    votes








    1 Answer
    1






    active

    oldest

    votes









    active

    oldest

    votes






    active

    oldest

    votes









    4












    $begingroup$

    We have $frac12 (partial_muA^mu)^2 = frac12 (partial_alpha A^alpha)(partial_betaA^beta)= frac12 (partial_alpha A_sigma) eta^sigmaalpha(partial_betaA_rho) eta^rhobeta$ so the derivative w.r.t. $partial_mu A_nu$ is



    $$frac12delta_alpha^mu delta_sigma^nu eta^sigmaalpha(partial_betaA_rho) eta^rhobeta+frac12(partial_alpha A_sigma) eta^sigmaalphadelta_beta^mu delta_rho^nu eta^rhobeta= frac12 eta^munu (partial_beta A^beta)+frac12 (partial_alphaA^alpha) eta^munu = (partial_rhoA^rho)eta^munu $$



    where I've freely labeled and relabeled dummy indices.






    share|cite|improve this answer









    $endgroup$

















      4












      $begingroup$

      We have $frac12 (partial_muA^mu)^2 = frac12 (partial_alpha A^alpha)(partial_betaA^beta)= frac12 (partial_alpha A_sigma) eta^sigmaalpha(partial_betaA_rho) eta^rhobeta$ so the derivative w.r.t. $partial_mu A_nu$ is



      $$frac12delta_alpha^mu delta_sigma^nu eta^sigmaalpha(partial_betaA_rho) eta^rhobeta+frac12(partial_alpha A_sigma) eta^sigmaalphadelta_beta^mu delta_rho^nu eta^rhobeta= frac12 eta^munu (partial_beta A^beta)+frac12 (partial_alphaA^alpha) eta^munu = (partial_rhoA^rho)eta^munu $$



      where I've freely labeled and relabeled dummy indices.






      share|cite|improve this answer









      $endgroup$















        4












        4








        4





        $begingroup$

        We have $frac12 (partial_muA^mu)^2 = frac12 (partial_alpha A^alpha)(partial_betaA^beta)= frac12 (partial_alpha A_sigma) eta^sigmaalpha(partial_betaA_rho) eta^rhobeta$ so the derivative w.r.t. $partial_mu A_nu$ is



        $$frac12delta_alpha^mu delta_sigma^nu eta^sigmaalpha(partial_betaA_rho) eta^rhobeta+frac12(partial_alpha A_sigma) eta^sigmaalphadelta_beta^mu delta_rho^nu eta^rhobeta= frac12 eta^munu (partial_beta A^beta)+frac12 (partial_alphaA^alpha) eta^munu = (partial_rhoA^rho)eta^munu $$



        where I've freely labeled and relabeled dummy indices.






        share|cite|improve this answer









        $endgroup$



        We have $frac12 (partial_muA^mu)^2 = frac12 (partial_alpha A^alpha)(partial_betaA^beta)= frac12 (partial_alpha A_sigma) eta^sigmaalpha(partial_betaA_rho) eta^rhobeta$ so the derivative w.r.t. $partial_mu A_nu$ is



        $$frac12delta_alpha^mu delta_sigma^nu eta^sigmaalpha(partial_betaA_rho) eta^rhobeta+frac12(partial_alpha A_sigma) eta^sigmaalphadelta_beta^mu delta_rho^nu eta^rhobeta= frac12 eta^munu (partial_beta A^beta)+frac12 (partial_alphaA^alpha) eta^munu = (partial_rhoA^rho)eta^munu $$



        where I've freely labeled and relabeled dummy indices.







        share|cite|improve this answer












        share|cite|improve this answer



        share|cite|improve this answer










        answered Mar 18 at 5:13









        DwaggDwagg

        619113




        619113



























            draft saved

            draft discarded
















































            Thanks for contributing an answer to Physics Stack Exchange!


            • Please be sure to answer the question. Provide details and share your research!

            But avoid


            • Asking for help, clarification, or responding to other answers.

            • Making statements based on opinion; back them up with references or personal experience.

            Use MathJax to format equations. MathJax reference.


            To learn more, see our tips on writing great answers.




            draft saved


            draft discarded














            StackExchange.ready(
            function ()
            StackExchange.openid.initPostLogin('.new-post-login', 'https%3a%2f%2fphysics.stackexchange.com%2fquestions%2f467109%2fapplying-the-euler-lagrange-equations-to-maxwells-theory%23new-answer', 'question_page');

            );

            Post as a guest















            Required, but never shown





















































            Required, but never shown














            Required, but never shown












            Required, but never shown







            Required, but never shown

































            Required, but never shown














            Required, but never shown












            Required, but never shown







            Required, but never shown







            Popular posts from this blog

            Adding axes to figuresAdding axes labels to LaTeX figuresLaTeX equivalent of ConTeXt buffersRotate a node but not its content: the case of the ellipse decorationHow to define the default vertical distance between nodes?TikZ scaling graphic and adjust node position and keep font sizeNumerical conditional within tikz keys?adding axes to shapesAlign axes across subfiguresAdding figures with a certain orderLine up nested tikz enviroments or how to get rid of themAdding axes labels to LaTeX figures

            Luettelo Yhdysvaltain laivaston lentotukialuksista Lähteet | Navigointivalikko

            Gary (muusikko) Sisällysluettelo Historia | Rockin' High | Lähteet | Aiheesta muualla | NavigointivalikkoInfobox OKTuomas "Gary" Keskinen Ancaran kitaristiksiProjekti Rockin' High